1. Suppose L is a regular language, and M = (Q, Σ, δ, q0, A) is a deterministic finite state machine such that L(M) = L. Prove that if |Q| = 2 then one of the following hold: (i)L=∅(ii)ε∈L,or(iii)∃a∈Σsuch that a∈L. 

"Get 15% discount on your first 3 orders with us"
Use the following coupon
FIRST15

Order Now